Nth digit of PI

Randall Hopper aa8vb at yahoo.com
Thu Jul 6 08:38:44 EDT 2000


Andrew Dalke:
 |I've seen the proof that e is irrational, but don't recall the details.
 |The sketch you outlined isn't sufficient.  Consider
 |
 |  sum(n=1..inf, 1/n^2) = 1 + 1/2 + 1/4 + 1/8 + ... = 2
 |
 |For any finite N in the sum 1..N, the denominator is infinitely large,
 |but the infinite sum is an integer.

Good point.  Thanks for the correction.

-- 
Randall Hopper
aa8vb at yahoo.com




More information about the Python-list mailing list